SlideShare a Scribd company logo
1 of 157
Objective Structured Practical
Examination (OSPE)
Md. Anisur Rahman
Professor & HOD (Ophthalmology)
DMC
7/1/2020 anjumk38dmc@gmail.com 1
OSPE: 01
Preparation of Methylprednisolone from this one gram vial with
this supplied materials: for the patient of RBN.
• One gram Methylprednisolone,
• 10 ml distilled water,
• 500 mg normal saline,
• syringe,
• Butterfly needle
7/1/2020 anjumk38dmc@gmail.com 2
A Wearing of the gloves 2.0
B Mix 10 ml distilled water in the vial 2,0
C Discard 300 ml normal saline 2,0
D Add this 10 ml medicine with the 200 ml normal
saline
2.0
7/1/2020 anjumk38dmc@gmail.com 3
OSPE: 02. History taking of R.P
• Greetings & self introduction.------------------------------------------0.25
• What is the duration of dimness of vision? --------------------------1.0
• Is the dimness of vision slowly progressive? ---------------------- 1.0
• Was it started with night vision problem? ----------------------------1.0
• Family history of RP. If yes, where they examine? ---------------- 1.0
• H/O consanguine marriage of parent---------------------------------1.0
• History of drug intake -------------------------------------------------- 0.5
7/1/2020 anjumk38dmc@gmail.com 4
• History of trauma. --------------------------------------------------0.5
• H/O hearing disorder, ataxia, nystagmus,--------------------- 0.75
• H/O mental retardation ---------------------------------------------1.0
• H/O heart disease? --------------------------------------------------0.5
• H/O hypogenitalism, obesity----------------------------------- 0.75
• H/O diarrhea,, skeletal deformity.--------------------------------- 0.5
• Thanks’ to the patient.-----------------------------------------------0.25
7/1/2020 anjumk38dmc@gmail.com 5
OSPE: 3. Your patient need FFA let you counsel the
patient
Greetings -------------------------------------------------------------0.5
explanation of procedure:
• Inj. Na Fluoride -----------------------------------------------------1.0
• Taking of picture ---------------------------------------------------1.0
Prerequisite:
• Dilated pupil --------------------------------------------------------1.0
• Renal function test -------------------------------------------------1.0
• Any hypersensitivity of Fluoride --------------------------------1.0
7/1/2020 anjumk38dmc@gmail.com 6
Possible side effects:
• Nausea/vomiting ------------------------------------------------------ 1.0
• Yellow urine ----------------------------------------------------------- 1.0
• Anaphylaxis /syncope ------------------------------------------------ 1.0
Talk about cost -------------------------------------------------------- 1.0
Thanks & Feedback --------------------------------------------------- 0.5
7/1/2020 anjumk38dmc@gmail.com 7
OSPE: 4
Pain that persists for more than one month after other signs and
symptoms disappear
a) What is the probable diagnosis?
b) Which age group suffers more?
c) What are the characteristic of pain? Mention 3
d) Why some of the patient suffered from depression?
e) What is the most devastating even life-threatening condition?
7/1/2020 anjumk38dmc@gmail.com 8
a) Post-herpetic neuralgia-----------------------------------------------2
b) Above 70 year.----------------------------------------------------------1
c) Pain may be
• constant or intermittent, ---------------------------------------------1
• worse at night and -----------------------------------------------------1
• aggravated by minor stimuli, touch and heat---------------------1
d) Neuralgia can impair the (QOL), and may lead to depression-1
Patient may commit suicide.--------------------------------------------1
7/1/2020 anjumk38dmc@gmail.com 9
OSPE: 5
• During Phacoemulsification of a 55-year-old diabetic
patient, nucleus dropped while chopping nucleus.
• Write a referral letter to respective specialist for further
management:
7/1/2020 anjumk38dmc@gmail.com 10
1) Registration number 1.0
2) Date 0.5
3) Identify of the referred doctor 0.5 x 3 1.5
• Full name
• Designation
• Address
4) Subject: Dropped nucleus during phaco 1.0
5) Greetings 0.5
7/1/2020 anjumk38dmc@gmail.com 11
6) Patient information 8 x 0.5 4.0
• Name
• Age
• Statement of the present condition
• When nucleus dropped date
• Description of dropped nucleus
• Vitrectomy done/not done
• IOL implanted/not implanted
• Present medication receiving
7/1/2020 anjumk38dmc@gmail.com 12
7) Thanks’ 0.5
8) Identity of referring doctor 0.25 x 4 1.0
• Full name
• Designation
• Address
• Signature
7/1/2020 anjumk38dmc@gmail.com 13
OSPE: 6
When you are a trainee in a tertiary center you receive a
physical assault patient suffering from rupture globe (sclera-
corneal injury) right eye. VA is PL+, PR. Now you receive a
letter from court to give an injury certificate. write it.
7/1/2020 anjumk38dmc@gmail.com 14
1) Admission Reg No: -----------------------------------------------1.0
2) Date & Time of Examination: ----------------------------------0.50
3) Name of the Patient: --------------------------------------------0.50
4) Age------------------------------------------------------------------0.50
5) Sex------------------------------------------------------------------0.25
6) C/O------------------------------------------------------------------0.50
7) Religion: Islam----------------------------------------------------0.25
7/1/2020 anjumk38dmc@gmail.com 15
8) Occupation-------------------------------------------------------0.25
9) Address----------------------------------------------------------0.25
10)Name of the Identifier----------------------------------------0.50
11)Injury note: -----------------------------------------------------1.0
12)VAR---------------------------------------------------------------0.50
13)VAL---------------------------------------------------------------0.50
7/1/2020 anjumk38dmc@gmail.com 16
14) Slit lamp examination---------------0.25 x 7 ---------------1.75
• Eyelids
• Conjunctiva
• Cornea
• AC
• Lens
• Pupil
• Fundus
7/1/2020 anjumk38dmc@gmail.com 17
15)Nature of weapon----------------------------------------------0.25
16)Nature of trauma-----------------------------------------------0.50
17)Comment---------------------------------------------------------0.25
18)Signature & Date------------------------------------------------0.25
19)(Seal)--------------------------------------------------------------0.25
20)Counter sign------------------------------------------------------0.25
• TOTAL------
7/1/2020 anjumk38dmc@gmail.com 18
OSPE: 7
• A 35 years old patient comes to you for Distance vision
and floaters on his Right eye. After taking history
ocular and systemic examination and investigations
you confirm the patient is suffering from HIV infection
and CMV retinitis and CD4 count is <50. You decide to
give HAART & Oral Valganciclovir. Counsel the patient.
7/1/2020 anjumk38dmc@gmail.com 19
1) Greetings
2) Give idea about his systemic disease-----------------------------------1.0
3) Give idea about his ocular disease--------------------------------------1.0
4) Explain that his ocular disease is due to his systemic disease-----1.0
5) Needs life style change----------------------------------------------------1.0
6) Needs consultation with physician--------------------------------------1.0
7) Treatment options of systemic disease---------------------------------1.0
8) Treatment option of ocular disease-------------------------------------1.0
7/1/2020 anjumk38dmc@gmail.com 20
9) Duration of treatment----------------------------------------------------0.5
10)Complication of treatment----------------------------------------------0.5
11) Give idea that treatment slow the systemic disease progression but
not cure absolutely--------------------------------------------------------0.5
12)Complication if treatment not taken-----------------------------------0.5
13)Follow up and screening------------------------------------------------0.25
14)Recurrence of ocular disease may occur-----------------------------0.25
15)Thanks’---------------------------------------------------------------------0.25
7/1/2020 anjumk38dmc@gmail.com 21
OSPE: 8
• Counsel a patient 65 years old, who is suffering from
glaucoma and he has also developed cataract with anti-
glaucoma eye drop his Intra Ocular Pressure (IOP) is 17
mm of Hg. Counsel the patient regarding your
treatment plan
7/1/2020 anjumk38dmc@gmail.com 22
1 Greetings
2 Telling about
What is cataract?
What is glaucoma?
3 What are the options of management
Cataract surgery followed by topical medication
Combined phaco trab / Combined SICS trab
Trabeculectomy followed by cataract surgery
7/1/2020 anjumk38dmc@gmail.com 23
4 Consequence of only cataract surgery
A Post-operative hike of IOP
B Medication should be continued for life long
C Glaucomatous damage can be progress
D Future glaucoma surgery may be difficult
E Trab surgery only and its disadvantage
F Advantage of combined surgery
5 Counseling about complications of surgeryFeedback from the patient
Thanks’ to the patient
7/1/2020 anjumk38dmc@gmail.com 24
OSPE: 9A man of 45 year old, refraction done by a junior resident, he
done refraction from 1 meter distance & his finding was as below
1) How many mistakes he has been committed?
2) And what are the mistakes
7/1/2020 anjumk38dmc@gmail.com 25
A. Four mistakes are there
B
a) In right axis will be 1800
b) In left eye power will be -1.0 Dsph
c) Addition will be + +1.50
d) IPD: 70/66 mm
7/1/2020 anjumk38dmc@gmail.com 26
OSPE: 10. Measure IOP with the tonometer with GAT
7/1/2020 anjumk38dmc@gmail.com 27
7/1/2020 anjumk38dmc@gmail.com 28
7/1/2020 anjumk38dmc@gmail.com 29
OSPE: 11. Counseling the parents of a child who may need
cycloplegic refractive correction (with Atropine 1% eye drop)
• Greetings to the parents ------------------------------------------0.25
• Explain the refractive error. --------------------------------------1.0
• Explain the option of treatment. -------------------------------1.0
• Explain what if not treated ------------------------------------- 1.0
• Explain cycloplegic refraction ---------------------------------- 1.0
• Explain the how to apply cycloplegic --------------------------0.5
• Dosage of Atropine-------------------------------------------------0.5
• Explain side effect of cycloplegic & precaution ------------- 1.5
7/1/2020 anjumk38dmc@gmail.com 30
• Side effects of Atropine 1% eye drop
• Fever
• Vision will be blurring for 10 to 14 days
• Care & use of spectacles ----------------------------------------------0.75
• Any query from the parents ------------------------------------------0.75
• Regular follow up ---------------------------------------------------------0.75
• Ensure the use of spectacle --------------------------------------------0.75
• Thanks’ --------------------------------------------------------------------- 0.25
7/1/2020 anjumk38dmc@gmail.com 31
These two pictures represent two diseases.
A. Mention the names of the diseases
B. Both the diseases involve different structures of the eyeball but
when you get these picture
C. In fig: 1, Is it active or chronic stage
D. Why vision loss is there (Fig: 1)? Mention 2
OSPE: 12
7/1/2020 anjumk38dmc@gmail.com 32
A. Ocular toxoplasmosis & Sarcoidosis --------------------- 4.0
B. When uveal tissue involve ---------------------------------1.5
C. Active-------------------------------------------------------------1.5
D. (Any 2) ---------------------------------------------------------3.0
• macular inflammatory lesions /oedema,
• optic nerve involvement
• vascular occlusion
7/1/2020 anjumk38dmc@gmail.com 33
OSPE: 13
• An old man of 70 year came to you with the complaints of
repeated attacks (three times in last two months) of suddenly
falls in the ground without warning or loss of consciousness
associated with transient, bilateral blurred vision usually lasting
a few seconds, sometimes accompanied by flushing lights,
ataxia, vertigo, perioral numbness and hemisensory loss
associated with visual symptoms. There was no disc oedema.
7/1/2020 anjumk38dmc@gmail.com 34
1) What may be the probable cause?
2) Mention two differential diagnosis
3) Mention one clinical examination & one laboratory
investigation to rule out the diagnosis, beyond eye.
4) Two hematological investigations.
5) Two radiological investigations.
7/1/2020 anjumk38dmc@gmail.com 35
1) Vertebro-basilar artery insufficiency ----------------------- 2
2) Any two (1 x 2) ------------------------------------------------ 2
• Giant cell arteritis (GCA)
• Ischemic optic neuropathy
• Central retinal artery/vein occlusion
• Migraine with aura
7/1/2020 anjumk38dmc@gmail.com 36
3) BP in each arm to rule out subclavian steal syndrome-------1
3) ECG/24 hours Holter monitor to rule out dysrhythmia-------1
4)
• CBC ----------------------------------------------------------------1
• Serum cholesterol ------------------------------------------------- 1
5)
• MRA ------------------------------------------------------------ 1
• Transcranial/vertebral artery Doppler -------------------- 1
7/1/2020 anjumk38dmc@gmail.com 37
Vertebrobasilar Insufficiency
Aetiopathogenesis: This is a vasculopathic disease affecting the
vertebrobasilar arterial supply. It manifests with symptoms due
to ischaemia of the brainstem and occipital cortex.
Risk factors include diabetes mellitus, hypertension,
hyperlipidaemia and cervical spondylosis
Ref: Parson’s 518p (22 nd edition)
7/1/2020 anjumk38dmc@gmail.com 38
Clinical features:
These include:
 episodes of transient blurred vision occurring bilaterally,
 lasting a few seconds to a few minutes which are
 sometimes accompanied by flashing lights.
7/1/2020 anjumk38dmc@gmail.com 39
Other associated symptoms
 transient diplopia,
 ataxia,
 vertigo,
 dysarthria,
 perioral paraesthesia,
 dysphasia,
 hemiparesis or hemisensory loss.
7/1/2020 anjumk38dmc@gmail.com 40
history of
The patient may also give a history of drop attacks (sudden
episodes of falling to the ground without warning or loss of
consciousness).
However, the eyes are completely normal on examination
7/1/2020 anjumk38dmc@gmail.com 41
Differential Diagnosis
7/1/2020 anjumk38dmc@gmail.com 42
7/1/2020 anjumk38dmc@gmail.com 43
Investigation
• A complete work-up should be carried out which includes all the
tests as for carotid occlusive disease,
• ECG and
• 24-hour Holter monitoring to rule out sick sinus syndrome and
ventricular ectopics,
7/1/2020 anjumk38dmc@gmail.com 44
• MRA or transcranial and vertebral Doppler ultrasound to
evaluate the posterior cerebral blood flow.
• X-rays of the cervical spine to rule out compressive disease of
the cervical spine (degenerative changes especially osteophytes
encroaching on the arterial foramina) are also required.
7/1/2020 anjumk38dmc@gmail.com 45
• A patient of 75-year came to you with the complain of vertigo
accompanied by double vision graying of vision, and blurred
vision, The patient gave history of suddenly become weak at the
knee and crumple a fall that lead to significant head injury.
There was no disc edema.
7/1/2020 anjumk38dmc@gmail.com 46
1) What may be the probable cause? And what are the differential
diagnoses?------------------------ 7
2) What are the clinical examination & bed side investigation to
rule out the diagnosis ------------- 8
3) What are the laboratory investigations you do to rule out the
others and why----------------------10
7/1/2020 anjumk38dmc@gmail.com 47
OSPE: 14
A lady-60–year-old came to you with the complaints of her
eyelash are lengthening, thickening and hyper pigmentation
after using an eye drop which you prescribed six months earlier.
Overdose of this drop may causes raised of intraocular pressure.
7/1/2020 anjumk38dmc@gmail.com 48
1) What is the name of the eye drop?
2) In which disease it is used?
3) What is the mechanism action of the medicine?
4) What is its dose schedule?
7/1/2020 anjumk38dmc@gmail.com 49
5) Name some drops of this group.
6) If there is no side effect, you have to stop the medication
before cataract surgery. Why?
7) Which hyper pigmentation is reversible and which is not
7/1/2020 anjumk38dmc@gmail.com 50
1) Prostaglandin analogue--------------------------------------2.0
2) Glaucoma-------------------------------------------------------1.5
3) Enhancement of uveoscleral aqueous outflow, although
increased trabecular outflow facility-----------------------1.5
4) Once in a day---------------------------------------------------1.0
7/1/2020 anjumk38dmc@gmail.com 51
5a) Latanaprost-------------------------------------------------------0.5
5b) Travaprost--------------------------------------------------------0.5
5c) Bimataprost-------------------------------------------------------0.5
5d) Tafluprost----------------------------------------------------------0.5
6) To prevent cystoid macular oedema----------------------------1.0
7) Hyper pigmentation of periocular skin is common but reversible.
Irreversible iris hyper pigmentation--------------0.5 + 0.5-----1.0
7/1/2020 anjumk38dmc@gmail.com 52
OSPE: 15
A-lady of -25-year old newly married came to you with
the complaints of non-specific headache worsening in
morning, diffuse type exacerbated by the Valsalva
maneuver. Take the relevant history for diagnosis.
7/1/2020 anjumk38dmc@gmail.com 53
7/1/2020 anjumk38dmc@gmail.com 54
7/1/2020 anjumk38dmc@gmail.com 55
7/1/2020 anjumk38dmc@gmail.com 56
OSPE: 16
Prepare Injection Amikacin for Intravitreal
7/1/2020 anjumk38dmc@gmail.com 57
7/1/2020 anjumk38dmc@gmail.com 58
7/1/2020 anjumk38dmc@gmail.com 59
OSPE: 17
A 7 year child has come to you by his parents with decreased
Visual acuity in both eyes. You have noticed that the patient is
irritable, with poor self- image. O/E You have found bruises over
hand. Ocular examination shows sub conjunctival hemorrhage
7/1/2020 anjumk38dmc@gmail.com 60
1) What may be the diagnosis?
2) Mention 2 posterior segments finding you look for.
3) Mention one systemic laboratory investigation
4) What is the treatment plan?
7/1/2020 anjumk38dmc@gmail.com 61
ANSWER & MARKING SCHEME
1) Battered baby syndrome 3
2) Multiple layer retinal hemorrhage and RAP 2
3) MRI of Brain, 2
4) Combined approach of pediatrician, ophthalmologist and
neurologist. 3
• TOTAL 10
7/1/2020 anjumk38dmc@gmail.com 62
OSPE: 18
Measure tear film breakup time:
(supplied materials: Fluorescein strip, slit lamp
7/1/2020 anjumk38dmc@gmail.com 63
7/1/2020 anjumk38dmc@gmail.com 64
7/1/2020 anjumk38dmc@gmail.com 65
OSPE:
19
7/1/2020 anjumk38dmc@gmail.com 66
1) Comment on the grey scale reading of the field.
2) Is the field reliable? Why?
3) What does 24-2 signify?
4) What is the testing strategy used in this patient?
5) Name 3 causes for such a field defect.
7/1/2020 anjumk38dmc@gmail.com 67
1) The grey scale reading of the right eye field indicates darker tone
occupying whole superior and inferior field only sparing central field
(Tubular field) 2.0
2) The field is reliable because the fixation loss, false positive and false
negative errors are within normal limits. 2.0
3) 24 signify temporal/central 24 degree and 2 indicates on either side
of the vertical and horizontal meridian points are tested. 1.5
7/1/2020 anjumk38dmc@gmail.com 68
• 4) The strategy used is Swedish interactive testing algorithm standard
format. 1.5
• 5) Any 3 3 x 1 3.0
a) Advanced Glaucoma (POAG),
b) Advanced RP,
c) After PRP,
d) CRAO with patent cilioretinal artery
• TOTAL 10
7/1/2020 anjumk38dmc@gmail.com 69
With SITA strategies, false negatives or false positives over about
15% should probably be regarded as highly significant, and with
full-threshold strategies, fixation losses over 20% and false
positives or negatives over 33%. In patients who consistently fail
to achieve good reliability it may be useful to switch to a
suprathreshold strategy or kinetic perimetry.
7/1/2020 anjumk38dmc@gmail.com 70
False positives are usually assessed by decoupling a stimulus
from its accompanying sound. If the sound alone is presented and
the patient still responds, a false positive is recorded. With a high
false-positive score the grey scale printout appears abnormally
pale. In SITA testing, false positives are estimated based on the
response time
7/1/2020 anjumk38dmc@gmail.com 71
• False negatives are registered by presenting a stimulus much
brighter than threshold at a location where the threshold has
already been determined. If the patient fails to respond, a false
negative is recorded. A high false-negative score indicates
inattention, tiredness or malingering, but is occasionally an
indication of disease severity rather than unreliability. The grey
scale printout in individuals with high false-negative responses
tends to have a clover leaf shape
7/1/2020 anjumk38dmc@gmail.com 72
7/1/2020 anjumk38dmc@gmail.com 73
• What is a Humphrey 24-2 and 30-2 visual field test?
• 24-2: Measures 24 degrees temporally and 30 degrees nasally
and tests 54 points. Used for neuro-ophthalmic conditions and
general screening as well as early detection of glaucoma. 30-2:
Measures 30 degrees temporally and nasally and tests 76 points.
Used for general screening, early glaucoma and neurological
conditions
7/1/2020 anjumk38dmc@gmail.com 74
OSPE: 20
The patient Mr. X 50 years old chooses this frame for
his spectacle. You measure the horizontal IPD of Mr. X
by using these instruments, which correspondence with
the spect frame
7/1/2020 anjumk38dmc@gmail.com 75
1 By using marker pen, mark the Centre of Bridge of
Spectacle on the front surface.
2 Put the frame on wearer’s face. Seat/stand directly in
front of the subject at the same level.
3 While the subject is looking at your left eye directly, put
the light of pen torch on wearer’s right eye and mark the
center of wearer’s right pupil (light reflex at the center
of the pupil) on the demo lens with a fine pen.
7/1/2020 anjumk38dmc@gmail.com 76
4 Repeat the same procedure on the wearer’s left eye.
5 Remove the frame, now measure the horizontal distance between
the ‘Mark’ on demo lens and center of the bridge of the frame.
6 The distance is measured in millimeter both for right and left lens
separately indicates the monocular CD. It is written in the form
33/32. The addition of two numbers should equal to binocular
IPD. So the B/O IPD is 65 mm.
7/1/2020 anjumk38dmc@gmail.com 77
What will be the visual field if damage occurs in 1,
3, 4 & 5?
OSPE: 21
7/1/2020 anjumk38dmc@gmail.com 78
1) Bitemporal hemianopia 2
2) Junctional scotoma 2
3) Upper temporal quadratic defect due to pituitary defect 2
4) Lower temporal quadratic defects due to craniopharyngioma 2
7/1/2020 anjumk38dmc@gmail.com 79
OSPE: 22
A lady of 33 year old, using spectacle since childhood, now she
came to you by heard the news of miracle can be done with
laser, which will give her life easy and she will be get rid of her
glass. So, you done some pre LASIK investigations, and result is
as followed.
7/1/2020 anjumk38dmc@gmail.com 80
• Topography
• Topography shows most of the colour in the center of the cornea
is hot colour.
• Topography also shows: Sim K1 in R/E is 47.50. Sim K2 50.50
• Sim k1 in L/E is 45.00. Sim K2 48.75
7/1/2020 anjumk38dmc@gmail.com 81
• Applanation Tonometry:
• R/E 15 mm of Hg. L/E 9 mm of Hg
• C:D ratio is
• 0.7 In R/E and 0.4 in L/E
• Schirmer test
• 2: 5 mm in B/E
Now, you have to counsel the lady about her desires:
7/1/2020 anjumk38dmc@gmail.com 82
• Rapport building--------------------------------------------0.25
• Tell about her topography---------------------------------3.0
• Cornea shows thinner in the apical portion so LASIK may not be
done
• Below 450 µm thickness is contraindicated.
• Tell about the Applanation Tonometer---------------------------1.5
• IOP change is significant (diff between 2 eyes)
7/1/2020 anjumk38dmc@gmail.com 83
• During the insertion of suction cup IOP may increase above 65
mm of Hg so there may be chance of vision reduction.
• C:D ratio is significant------------------------------------------1.0
• So, you may be a patient of POAG
• To diagnose the POAG we should do VFA & OCT
• POAG is a silent killer of vision.
7/1/2020 anjumk38dmc@gmail.com 84
• You are suffering from dry eye, LASIK will also increase dryness----1.0
• You can use RGP contact lens-----------------------------------1.0
• Tell her about the advantage & disadvantage of RGP lens as well as
cost-----------------------------1.0
• Other option is clear lens extraction--------------------------------1.0
• Thanks’ to the patient-----------------------------------------------0.25
• TOTAL ---------------------------------------------------------------10
7/1/2020 anjumk38dmc@gmail.com 85
OSPE: 23
• It is an idiopathic multisystem disorder. One of the most
common systemic associations of uveitis. It can affect essentially
any organ system. It more frequently (10:1) affects patients of
black than white ethnicity but is more common in colder
climates. Female more commonly affected than male peak age is
20-50 years.
7/1/2020 anjumk38dmc@gmail.com 86
1) What is the name of the disease?
2) Name the 3 major organs it affected.
3) Write 2 anterior segment findings
4) Write 3 posterior segment findings
7/1/2020 anjumk38dmc@gmail.com 87
1) Sarcoidosis------------------------------------------- 2.0
2) 3x1-------------------------------------------------------- 3.0
• a) Lung b) Skin c) Eye.
3) Any two----------2x1---------------------------------------2.0
• a) Conjunctival granuloma
• b) Lacrimal gland involvement/dry eye
• c) Acute or chronic uveitis
7/1/2020 anjumk38dmc@gmail.com 88
4) Any 3-----------------3x1--------------------------------------- 3.0
• a) Periphlebitis
• b) Choroidal infiltrates
• c) Multifocal Choroiditis
• d) Retinal granuloma
• e) Peripheral retinal neovascularization
• f) Optic nerve involvement.
• TOTAL---------------------------------------------------------- 10
7/1/2020 anjumk38dmc@gmail.com 89
OSPE: 24
• It is an autoimmune disorder characterized by multisystem
inflammation with the generation of autoantibodies. Although
the specific cause is unknown. Female at child bearing age is
commonly affected. Antibodies (dsDNA) are relatively
specific. Renal involvement is common presenting feature.
7/1/2020 anjumk38dmc@gmail.com 90
1) What is the probable diagnosis?
2) Mention 4 hematological test
3) How many patient suffering from ocular involvement?
4) Which one is the most common ocular involvement?
5) What are the most vision threatening complication? Mention 2
7/1/2020 anjumk38dmc@gmail.com 91
1) SLE
2) .Any 4
a) CBC with ESR
b) Serum creatinine
c) Complement levels
d) Liver function tests
e) Autoantibody tests
7/1/2020 anjumk38dmc@gmail.com 92
• 3) One third
• 4)
• The most common manifestation is keratoconjunctivitis sicca.
The most vision threatening are
a) retinal vasculitis and
b) optic neuritis/neuropathy.
7/1/2020 anjumk38dmc@gmail.com 93
OSPE: 25 (Counseling)
• A 50 year-old man with a painful right blind eye. Advice
regarding the various options for treating his painful eye
including enucleation or evisceration.
7/1/2020 anjumk38dmc@gmail.com 94
Suggestion: OSPE: 25
Suggestion:
Ascertain the severity of the pain and the patient's desire
of keeping an intact globe. If enucleation is contemplated,
consider the need for artificial eye service etc.
7/1/2020 anjumk38dmc@gmail.com 95
OSPE: 26 (Counseling)
• A 63 year-old man returns 4 weeks later for a follow up cataract
visit. Refraction shows that a wrong lens has been inserted
resulting in a hypermetropic shift. The patient is unhappy that
he could neither read nor see distance without glasses. Explain
to him what has happened and the various options open to him
including lens exchange and contact lenses.
7/1/2020 anjumk38dmc@gmail.com 96
Suggestion: OSPE: 26
Find out how inconvenience it is for the patient. Whether the
patient was hypermetropic before the operation and what he had
been told about the final refraction prior to the operation? Explain
the various options open to him.
7/1/2020 anjumk38dmc@gmail.com 97
OSPE: 27 (Counseling)
A 35 year-old has had two failed corneal graft for heretic
corneal disease. The cornea is heavily vascularized. He likes to
have another corneal graft but you know there is little of a
successful outcome. Explain to him why you think a re-graft is
not appropriate.
7/1/2020 anjumk38dmc@gmail.com 98
Suggestion: OSPE: 27
Find out why the patient wants a re-graft and his understanding
about the success rate. Is he concerned about the cosmetic effect
of a leucoma? In which case cosmetic contact lens may be
useful.)
7/1/2020 anjumk38dmc@gmail.com 99
OSPE: 28 (Counseling)
A 69 year-old woman is referred by her GP for possible cataract
operation because of very poor vision in both eyes. After
examining the patients, you discover that she has minimal
cataract and the poor vision is caused by bilateral disciform
macular degeneration. Explain to the patient why you think that
cataract is inappropriate.
7/1/2020 anjumk38dmc@gmail.com 100
Suggestion: OSPE: 28
• Explain to the patient what the problem is and why cataract
extraction is unhelpful. Use the analogy of a camera to describe
the eye for example: ' The lens of a camera is similar to the lens of
your eye and the film within the camera is similar to the back of
your eye. When the lens is damaged we can replace it with a new
one which is similar to a cataract operation. However, if the film
is scratchy, changing the lens may not alter the image captured on
the camera. The problem you have is similar to a scratchy film so
cataract operation will not improve your vision.' If there is a
model of the eye present, use it to illustrate her problem. Suggest
blind registration or partial sighted registration and low visual
aids.)
7/1/2020 anjumk38dmc@gmail.com 101
OSPE: 29 (Counseling)
A woman has a 2 year old child with bilateral retinoblastoma.
She is planning a second pregnancy and likes to know the risk of
a having another child with retinoblastoma.
7/1/2020 anjumk38dmc@gmail.com 102
Find out how much the patient knows about the conditions and the
inherited patterns. Is her partner aware of her desire of having a
second child? Is it possible for the partner to be present during the
consultation?)
7/1/2020 anjumk38dmc@gmail.com 103
OSPE: 30 (Scenario based)
• 71-year-old woman presents with the chief complaint of
distorted central vision. Funduscopic examination reveals the
presence of subretinal neovascularization and there are
depigmented areas in the macula. Distinct yellow-white lesions
are seen in the posterior pole surrounding the macula. The
patient reports wavy lines during Amsler grid testing.
7/1/2020 anjumk38dmc@gmail.com 104
Question
1) What is your probable diagnosis?
2) What are the layers it involves?
3) A highly magnified stereo-examination of the macula is
mandatory. What is the clinical examination?
4) What is the most sensitive investigative modality to detect?
5) How he will follow-up at home?
6) When vision is grossly diminished?
7/1/2020 anjumk38dmc@gmail.com 105
1) ARMD
2) choriocapillaris, Bruch’s membrane, retinal pigment
epithelium and photoreceptors.
3) The best clinical method is slit-lamp biomicroscopy using a 190
D or 178 D lens
4) OCT (Macular protocol)
7/1/2020 anjumk38dmc@gmail.com 106
5) Amsler grid testing
6) visual impairment may occur associated with a generalized
granularity and/or atrophy of the retinal pigment epithelium,
photoreceptors and choriocapillaris.
7/1/2020 anjumk38dmc@gmail.com 107
OSPE: 31 (Scenario based)
• A 50 year old man working at office has been complaining
diplopia. He also complaining deglutition difficulty at evening for
few days
1) What may be the cause?
2) Name 2 bed side test?
3) What is the confirmatory investigation?
4) What is the role of neuro imaging?
7/1/2020 anjumk38dmc@gmail.com 108
1) Myasthenia gravis
2) sleep test, ice pack test
3) Serum anti ACH-R ab
4) CXR/ CT scan to detect thymoma
7/1/2020 anjumk38dmc@gmail.com 109
Sleep test in myasthenia gravis
• The diagnosis is usually confirmed by a Tensilon test, which can
be complicated by cholinergic side effects that include
cardiopulmonary arrest. An alternative, the Sleep test, based on
the characteristic of myasthenia that symptoms and signs
worsen with fatigue and improve after a period of rest, is safe,
moderately sensitive, and specific. The diagnosis of myasthenia
can be confirmed by observing resolution of ptosis or
ophthalmoparesis immediately after a 30-minute period of
sleep; the reappearance of the myasthenic signs over the next 30
seconds to 5 minutes adds further confirmation.
7/1/2020 anjumk38dmc@gmail.com 110
OSPE: 32 (Scenario based)
A 40 year old man has got admitted in emergency department with
history of gun -shot injury one day back. On examination VA right eye
NPL, anterior & posterior segment reveals normal(kanski-873)
1) What may be the diagnosis?
2) What is the pathogenesis?
3) What is important clinical finding?
4) What is the treatment?
7/1/2020 anjumk38dmc@gmail.com 111
1) Traumatic optic neuropathy
2) Acceleration of optic nerve at optic canal where it attached to
dural sheath due to rupture of micro vascular supply
3) RAPD
4) Pulse dose IV corticosteroid.
7/1/2020 anjumk38dmc@gmail.com 112
OSPE: 33 (Scenario based)
A 4 year old child with severe bilateral congenital myogenic ptosis
with poor elevator function.
a) What will be his chin position?
b) If amblyopia is there, what may be the commonest reason?
c) If Ptosis is unilateral what will be the position of ptotic lid in down
gaze in comparison to normal eye?
d) What percent of congenital ptosis are associated with the Marcus
Gunn jaw-winking phenomenon 5%
7/1/2020 anjumk38dmc@gmail.com 113
a) will have a chin up head position
b) Refractive error
c) Above
d) 5%
7/1/2020 anjumk38dmc@gmail.com 114
OSPE: 34 (Scenario based)
It is the second most common retinal vascular disorder after
diabetic retinopathy.
a) What is the name of the diseases?
b) What is about vision?
c) What is the most common site of involvement? And why?
d) Sometimes there is persistent poor vision. What is the reason?
7/1/2020 anjumk38dmc@gmail.com 115
a) BRVO
b) If the central macula is involved consist of the sudden painless
onset of blurred vision and metamorphopsia. Peripheral
occlusion may be asymptomatic.
c) Superotemporal quadrant. The increased incidence in the
Superotemporal quadrant is thought to be due to increased arteriovenous
crossings in that quadrant.
d) Due to chronic macular edema
7/1/2020 anjumk38dmc@gmail.com 116
OSPE: 35 (Scenario based)
• A sudden painless monocular fall in vision. The pathogenesis is
believed to follow the principles of Virchow’s triad for
thrombogenesis, involving vessel damage, stasis and
hypercoagulability© Age is the most important factor, since over 90%
of cases occur in patients over the age of 55 years.
a) What is the probable diagnosis?
b) What are the risk factors? Mention 4.
c) What are the fundus features? 4
7/1/2020 anjumk38dmc@gmail.com 117
a) CRVO
b) Risk Factors (any: 4)
• Age
• Hypertension
• Hyperlipidaemia
• Diabetes mellitus
• Oral contraceptive pill
• Raised intraocular pressure
• Smoking
7/1/2020 anjumk38dmc@gmail.com 118
c)
i. Tortuosity and dilatation of all branches of the central retinal
vein,
ii. dot, blot and flame haemorrhages
iii. cotton wool spots,
iv. optic disc and macular oedema mild
7/1/2020 anjumk38dmc@gmail.com 119
OSPE: 36
What will be the image characteristic when object is at 2f1:
(Convex Lens)
a) Location
b) Orientation
c) Size
d) Type of image
7/1/2020 anjumk38dmc@gmail.com 120
a) Location: At 2 f2
b) Orientation: inverted
c) Size: Same
d) Type of image: Real
7/1/2020 anjumk38dmc@gmail.com 121
7/1/2020 anjumk38dmc@gmail.com 122
OSPE: 37
Write down the retinoscope
findings of the following glass
from 67 cm distance
OD: -0.75 Dsph/-0.75Dcyl 180
OS: Plano/-1.00 Dcyl 180
7/1/2020 anjumk38dmc@gmail.com 123
Write down the
retinoscope findings of the
following glass from one
meter distance
OD: -0.75 Dsph/-0.75Dcyl
180
OS: Plano/-1.00 Dcyl 180
OSPE: 38
A patient, who is suffering from cataract came for surgery but
uncontrolled diabetes, refer the patient to diabetologist with a referral
note
ANS: 38
A. Date & time: 29/04/2018
B. Time: 8-30 am.
C. To,
D. Assistant / Associate professor
E. Department of Endocrinology
F. DMCH
G. Dear Sir.
H. With due respect
I. I am the undersigned, like to inform you that patient Mr. X, 67 years old, hailing
from Manikganj suffering from bilateral Senile Mature Cataract (B/E) he is also
a patient of DM last 20 years, at present his blood sugar (2 hours ABF) is 15
Mmol/dl in spite of taking insulin regularly. He needs cataract surgery but
uncontrolled diabetes is contraindicated for his surgery.
• So, I will be obliged if your kind enough to give your valuable opinion
regarding his diabetes control
J. Thank you
K. Signature
L, Name of doctor
M. Designation
N. Institute & Department
Marking Scheme:
A. Date & time-----------------------------------------------------------------1
B. Addressing: Professor/Assoc prof/ Assitt Prof-------------------------1
C. Dear sir----------------------------------------------------------------------1
• Content:
D. Patient identification------------------------------------------------------1
E, From where-----------------------------------------------------------------1
F. Patient findings-------------------------------------------------------------1
G. What we want--------------------------------------------------------------1
H. Thank you-------------------------------------------------------------------1
I. Name & designation--------------------------------------------------------1
J. Signature---------------------------------------------------------------------1
TOTAL: -------------------------------------------------------------------10
OSPE: 39
A 20-year girl having uneventful DCR surgery under
L/A right side of the eye Prepare a discharge certificate
for the patient.
ANS: 39
Parameters Marks
A. Identification of
the patient
Name 0.5
Age 0.25
Gender 0.25
Address 0.5
Mobile No. 0.25
B. Operation
note
Date & time 0.25
Name of surgery 0.25
Indication of surgery 0.25
Name of anesthesia 0.5
Name of surgeon 0.5
C. Post-operative findings Condition of Incision area 0.5
Any discharge 0.5
Conjunctiva, eyelid, cornea 0.5
D. Post-operative treatment Systemic antibiotic 1.0
Systemic analgesic 0.5
Anti ulcerant 0.25
Topical antibiotic 0.5
E. Advice
No water to eye 0.5
Use dark glass 0.25
Regular use of medicine 0.25
Any problem come to doctor 0.25
Follow up 0.25
F. Identification of certificate
preparatory
Signature with date 0.25
Name of the doctor with
designation
0.5
Seal of the department 0.5
 TOTAL 10
OSPE: 40
A 60 years old male patient having uneventful phacoemulsification
with PC- IOL implantation under topical anesthesia in his right eye
Prepare a discharge certificate for the patient.
ANS: 40
Parameters Marks
A. Identification of the
patient
Name 0.5
Age 0.25
Gender 0.25
Address 0.5
Mobile No. 0.25
B. Operation note
Date & time 0.5
Name of surgery 0.5
Name of anesthesia 0.5
Name of surgeon 0.5
C. Post-operative findings Visual acuity 1.0
Anterior segment 1.0
Posterior segment 0.5
D. Post-operative
treatment
Topical antibiotic 0.5
Topical steroid 0.5
E. Advice
No water to eye 0.25
Use dark glass 0.25
Regular use of medicine 0.25
Any problem come to doctor 0.25
Follow up 0.25
F. Identification of
certificate preparatory
Signature with date 0.5
Name of the doctor with
designation
0.5
Seal of the department 0.5
TOTAL 10
OSPE: 41
A. Identify the instrument? Who
invented it?
B. What is function of the sleeve that
covers the tip of the internal tube?
C. Name 3 important parameters that
have to present during surgery
ANS: 41
A.
a) Phaco hand piece---------------------------------------------------------1
b) Charles David Kelman----------- +0.5 + 0.5 + 1.0 ------------------2
B. It protects the cornea, iris from transmitted heat energy by the
probe --------------------------------------------------------------------2
C.
a) Power ----------------------------------------------------------------------2
b) Vacuum --------------------------------------------------------------------2
c) Flow rate ------------------------------------------------------------------1
TOTAL---------------------------------------------------------------------10
These 3 pictures indicate post-
operative complication of cataract
surgery. What are the names? And
why they occur?
ANS: 42
 Fig: 1.
 Sunrise syndrome: -------------------------------------------------------- 1
 most commonly occurs due to the misplacement of the superior
haptic in the ciliary sulcus, while the inferior one is placed into the
capsular bag that allows the IOL to subluxated superiorly. ------------3
 Fig: 2.
 Sunset syndrome: ---------------------------------------------------------1
 May result from undetected anterior capsule rupture extending
inferiorly allowing the inferior haptic of PCIOL to escape through the
defects? ----------------------------------------------------------------------3
Fig: 3
Windshield wiper syndrome-----------------------------------------------1
When the overall size of the IOL is smaller it moves horizontally ---1
 TOTAL---------------------------------------------------------------------10
OSPE: 43
A. What is the white area around the
disc?
B. Does it usually create any
symptoms?
C. What change you may get in
visual field?
D. Can it disappear? If yes, when?
A. Medullated nerve fiber-------------------------------------------------2.5
B. No-------------------------------------------------------------------------2.5
C. There will be scotoma opposite to the nerve------------------------2.5
D. Yes. In optic atrophy or demyelinated diseases---------------------2.5
TOTAL--------------------------------------------------------------------10
OSPE: 44
Applying pressure patches and shields with following materials
• eye pad,
• eye shield,
• scissors,
• adhesive tape,
• alcohol pad.
ANS: 44
A. Apply a piece of adhesive tape, about 15 centimeters long, to the eye
pad---------------------------------------------------------------------------2
B. Ask the patient to close both eyes----------------------------------------2
C. Clean the forehead and zygoma with an alcohol pad to remove the
skin oils. This helps the tape stick to the skin--------------------------2
D. Fold one pad half, and position the eye pad diagonally over the closed lids
of the affected eye and tape firmly, but gently, to forehead and cheek----2
E. Apply a second and third piece of tape to ensure the pad lies flat----------1
F. Extra protection can be given by taping a shield over the pad --------------1
TOTAL------------------------------------------------------------------------------10
7/1/2020 anjumk38dmc@gmail.com 152
OSPE: 45
This is the feature of a
patient right eye, 15 days
after pterygium surgery
(surgery was done in bare
sclera technique followed by
MMC –C eye drop)
A. What is the feature now?
B. What is the most probable cause?
C. Mention 2 symptoms
D. What grievous complication impending to occur?
E. How can you manage it?
ANS: 45
A. Thinning of sclera----------------------------------------------------------2
B. Mitomycin-C---------------------------------------------------------------2
C. Severe pain, photophobia-------------------------------------------------2
D. Perforation of sclera-------------------------------------------------------2
E. Patch scleral graft----------------------------------------------------------2
TOTAL--------------------------------------------------------------------10
7/1/2020 anjumk38dmc@gmail.com 155
OSPE:46
7/1/2020 anjumk38dmc@gmail.com 156
A.What is the name of the tracing paper?
B.What is about RNFL thickness map?
C.What is about RNFL deviation map?
D.What is about TSNIT map?
E.What is about colour code?
F. What is your conclusion?
7/1/2020 anjumk38dmc@gmail.com 157
A.This is the printout paper of OCT optic nerve head and
RNFL of BE
B.RNFL thickness map normal
C.RNFL deviation map normal
D.TSNIT curve shows well maintained of double hump
pattern
E.The four quadrants show green colour and clock hour
map also shows green colours.
F. So suggestive of a normal OCT of ONH and RNFL

More Related Content

What's hot

What's hot (20)

Secondary open angle glaucoma
Secondary open angle glaucomaSecondary open angle glaucoma
Secondary open angle glaucoma
 
Embryology and anatomy of human lens
Embryology and anatomy of human lensEmbryology and anatomy of human lens
Embryology and anatomy of human lens
 
Fungal corneal ulcer
Fungal corneal ulcerFungal corneal ulcer
Fungal corneal ulcer
 
Congenital corneal disorders
Congenital corneal disordersCongenital corneal disorders
Congenital corneal disorders
 
Complications of cataract surgery
Complications of cataract surgeryComplications of cataract surgery
Complications of cataract surgery
 
Chemical injuries of the eye
Chemical injuries of the eyeChemical injuries of the eye
Chemical injuries of the eye
 
Cyclocryo
CyclocryoCyclocryo
Cyclocryo
 
Dry eye
Dry eye Dry eye
Dry eye
 
Corneal degeneration &amp; depos
Corneal degeneration &amp; deposCorneal degeneration &amp; depos
Corneal degeneration &amp; depos
 
Differential Diagnosis of Disc Edema
Differential Diagnosis of Disc EdemaDifferential Diagnosis of Disc Edema
Differential Diagnosis of Disc Edema
 
Blunt Ocular Trauma
Blunt Ocular TraumaBlunt Ocular Trauma
Blunt Ocular Trauma
 
neovascular glaucoma
neovascular glaucomaneovascular glaucoma
neovascular glaucoma
 
Trabeculectomy, trabeculotomy, goniotomy and their complications
Trabeculectomy, trabeculotomy, goniotomy and their complicationsTrabeculectomy, trabeculotomy, goniotomy and their complications
Trabeculectomy, trabeculotomy, goniotomy and their complications
 
CORNEAL DYSTROPHIES
CORNEAL DYSTROPHIESCORNEAL DYSTROPHIES
CORNEAL DYSTROPHIES
 
Hvf progession
Hvf progessionHvf progession
Hvf progession
 
Congenital nasolacrimal duct obstruction
Congenital nasolacrimal duct obstructionCongenital nasolacrimal duct obstruction
Congenital nasolacrimal duct obstruction
 
Cataracts in paediatric patients
Cataracts in paediatric patients Cataracts in paediatric patients
Cataracts in paediatric patients
 
Steroid Induced Glaucoma - Dr Shylesh B Dabke
Steroid Induced Glaucoma - Dr Shylesh B DabkeSteroid Induced Glaucoma - Dr Shylesh B Dabke
Steroid Induced Glaucoma - Dr Shylesh B Dabke
 
Ocular Manifestation of Diabetes Mellitus
Ocular Manifestation of Diabetes MellitusOcular Manifestation of Diabetes Mellitus
Ocular Manifestation of Diabetes Mellitus
 
Ramji pandey ppt vkc
Ramji pandey  ppt vkcRamji pandey  ppt vkc
Ramji pandey ppt vkc
 

More from Anisur Rahman

More from Anisur Rahman (20)

Hypertensive retinopathy
Hypertensive retinopathyHypertensive retinopathy
Hypertensive retinopathy
 
LASER
LASERLASER
LASER
 
Goldman Applanation Tonometer
Goldman Applanation TonometerGoldman Applanation Tonometer
Goldman Applanation Tonometer
 
Neuro-ophthalmology
Neuro-ophthalmologyNeuro-ophthalmology
Neuro-ophthalmology
 
Central tendency and dispersion
Central tendency and dispersionCentral tendency and dispersion
Central tendency and dispersion
 
Ophthalmoscope direct and indirect
Ophthalmoscope direct and indirectOphthalmoscope direct and indirect
Ophthalmoscope direct and indirect
 
Neuro ophthalmology
Neuro ophthalmologyNeuro ophthalmology
Neuro ophthalmology
 
Refractive error
Refractive error Refractive error
Refractive error
 
04 lecture Neuro-ophthalmology
04 lecture Neuro-ophthalmology04 lecture Neuro-ophthalmology
04 lecture Neuro-ophthalmology
 
04 prism
04 prism 04 prism
04 prism
 
06 lecture
06 lecture06 lecture
06 lecture
 
03 mirror and lens
03 mirror and lens03 mirror and lens
03 mirror and lens
 
03 lecture neuro
03 lecture neuro03 lecture neuro
03 lecture neuro
 
5th lecture on research methodology
5th lecture on research methodology5th lecture on research methodology
5th lecture on research methodology
 
02 lecture 16 april
02 lecture 16 april02 lecture 16 april
02 lecture 16 april
 
Second lecture neuro ophthalmology
Second lecture neuro ophthalmologySecond lecture neuro ophthalmology
Second lecture neuro ophthalmology
 
Sample and Sampling Technique 3rd Lecture
Sample and Sampling Technique 3rd LectureSample and Sampling Technique 3rd Lecture
Sample and Sampling Technique 3rd Lecture
 
Optics 09 april 2021
Optics 09 april 2021Optics 09 april 2021
Optics 09 april 2021
 
0 protocol
0 protocol 0 protocol
0 protocol
 
Ospe mbbs
Ospe mbbsOspe mbbs
Ospe mbbs
 

Recently uploaded

Difference Between Skeletal Smooth and Cardiac Muscles
Difference Between Skeletal Smooth and Cardiac MusclesDifference Between Skeletal Smooth and Cardiac Muscles
Difference Between Skeletal Smooth and Cardiac Muscles
MedicoseAcademics
 
Cara Menggugurkan Kandungan Dengan Cepat Selesai Dalam 24 Jam Secara Alami Bu...
Cara Menggugurkan Kandungan Dengan Cepat Selesai Dalam 24 Jam Secara Alami Bu...Cara Menggugurkan Kandungan Dengan Cepat Selesai Dalam 24 Jam Secara Alami Bu...
Cara Menggugurkan Kandungan Dengan Cepat Selesai Dalam 24 Jam Secara Alami Bu...
Cara Menggugurkan Kandungan 087776558899
 
Russian Call Girls In Pune 👉 Just CALL ME: 9352988975 ✅❤️💯low cost unlimited ...
Russian Call Girls In Pune 👉 Just CALL ME: 9352988975 ✅❤️💯low cost unlimited ...Russian Call Girls In Pune 👉 Just CALL ME: 9352988975 ✅❤️💯low cost unlimited ...
Russian Call Girls In Pune 👉 Just CALL ME: 9352988975 ✅❤️💯low cost unlimited ...
chanderprakash5506
 
👉 Chennai Sexy Aunty’s WhatsApp Number 👉📞 7427069034 👉📞 Just📲 Call Ruhi Colle...
👉 Chennai Sexy Aunty’s WhatsApp Number 👉📞 7427069034 👉📞 Just📲 Call Ruhi Colle...👉 Chennai Sexy Aunty’s WhatsApp Number 👉📞 7427069034 👉📞 Just📲 Call Ruhi Colle...
👉 Chennai Sexy Aunty’s WhatsApp Number 👉📞 7427069034 👉📞 Just📲 Call Ruhi Colle...
rajnisinghkjn
 

Recently uploaded (20)

Difference Between Skeletal Smooth and Cardiac Muscles
Difference Between Skeletal Smooth and Cardiac MusclesDifference Between Skeletal Smooth and Cardiac Muscles
Difference Between Skeletal Smooth and Cardiac Muscles
 
💞 Safe And Secure Call Girls Coimbatore🧿 6378878445 🧿 High Class Coimbatore C...
💞 Safe And Secure Call Girls Coimbatore🧿 6378878445 🧿 High Class Coimbatore C...💞 Safe And Secure Call Girls Coimbatore🧿 6378878445 🧿 High Class Coimbatore C...
💞 Safe And Secure Call Girls Coimbatore🧿 6378878445 🧿 High Class Coimbatore C...
 
Call Girls Bangalore - 450+ Call Girl Cash Payment 💯Call Us 🔝 6378878445 🔝 💃 ...
Call Girls Bangalore - 450+ Call Girl Cash Payment 💯Call Us 🔝 6378878445 🔝 💃 ...Call Girls Bangalore - 450+ Call Girl Cash Payment 💯Call Us 🔝 6378878445 🔝 💃 ...
Call Girls Bangalore - 450+ Call Girl Cash Payment 💯Call Us 🔝 6378878445 🔝 💃 ...
 
Call Girls Mussoorie Just Call 8854095900 Top Class Call Girl Service Available
Call Girls Mussoorie Just Call 8854095900 Top Class Call Girl Service AvailableCall Girls Mussoorie Just Call 8854095900 Top Class Call Girl Service Available
Call Girls Mussoorie Just Call 8854095900 Top Class Call Girl Service Available
 
ANATOMY AND PHYSIOLOGY OF REPRODUCTIVE SYSTEM.pptx
ANATOMY AND PHYSIOLOGY OF REPRODUCTIVE SYSTEM.pptxANATOMY AND PHYSIOLOGY OF REPRODUCTIVE SYSTEM.pptx
ANATOMY AND PHYSIOLOGY OF REPRODUCTIVE SYSTEM.pptx
 
Circulatory Shock, types and stages, compensatory mechanisms
Circulatory Shock, types and stages, compensatory mechanismsCirculatory Shock, types and stages, compensatory mechanisms
Circulatory Shock, types and stages, compensatory mechanisms
 
Bhawanipatna Call Girls 📞9332606886 Call Girls in Bhawanipatna Escorts servic...
Bhawanipatna Call Girls 📞9332606886 Call Girls in Bhawanipatna Escorts servic...Bhawanipatna Call Girls 📞9332606886 Call Girls in Bhawanipatna Escorts servic...
Bhawanipatna Call Girls 📞9332606886 Call Girls in Bhawanipatna Escorts servic...
 
7 steps How to prevent Thalassemia : Dr Sharda Jain & Vandana Gupta
7 steps How to prevent Thalassemia : Dr Sharda Jain & Vandana Gupta7 steps How to prevent Thalassemia : Dr Sharda Jain & Vandana Gupta
7 steps How to prevent Thalassemia : Dr Sharda Jain & Vandana Gupta
 
Call Girls in Lucknow Just Call 👉👉8630512678 Top Class Call Girl Service Avai...
Call Girls in Lucknow Just Call 👉👉8630512678 Top Class Call Girl Service Avai...Call Girls in Lucknow Just Call 👉👉8630512678 Top Class Call Girl Service Avai...
Call Girls in Lucknow Just Call 👉👉8630512678 Top Class Call Girl Service Avai...
 
ANATOMY AND PHYSIOLOGY OF RESPIRATORY SYSTEM.pptx
ANATOMY AND PHYSIOLOGY OF RESPIRATORY SYSTEM.pptxANATOMY AND PHYSIOLOGY OF RESPIRATORY SYSTEM.pptx
ANATOMY AND PHYSIOLOGY OF RESPIRATORY SYSTEM.pptx
 
Cara Menggugurkan Kandungan Dengan Cepat Selesai Dalam 24 Jam Secara Alami Bu...
Cara Menggugurkan Kandungan Dengan Cepat Selesai Dalam 24 Jam Secara Alami Bu...Cara Menggugurkan Kandungan Dengan Cepat Selesai Dalam 24 Jam Secara Alami Bu...
Cara Menggugurkan Kandungan Dengan Cepat Selesai Dalam 24 Jam Secara Alami Bu...
 
Call Girls Service Jaipur {9521753030 } ❤️VVIP BHAWNA Call Girl in Jaipur Raj...
Call Girls Service Jaipur {9521753030 } ❤️VVIP BHAWNA Call Girl in Jaipur Raj...Call Girls Service Jaipur {9521753030 } ❤️VVIP BHAWNA Call Girl in Jaipur Raj...
Call Girls Service Jaipur {9521753030 } ❤️VVIP BHAWNA Call Girl in Jaipur Raj...
 
Call Girls Wayanad Just Call 8250077686 Top Class Call Girl Service Available
Call Girls Wayanad Just Call 8250077686 Top Class Call Girl Service AvailableCall Girls Wayanad Just Call 8250077686 Top Class Call Girl Service Available
Call Girls Wayanad Just Call 8250077686 Top Class Call Girl Service Available
 
Call 8250092165 Patna Call Girls ₹4.5k Cash Payment With Room Delivery
Call 8250092165 Patna Call Girls ₹4.5k Cash Payment With Room DeliveryCall 8250092165 Patna Call Girls ₹4.5k Cash Payment With Room Delivery
Call 8250092165 Patna Call Girls ₹4.5k Cash Payment With Room Delivery
 
Lucknow Call Girls Service { 9984666624 } ❤️VVIP ROCKY Call Girl in Lucknow U...
Lucknow Call Girls Service { 9984666624 } ❤️VVIP ROCKY Call Girl in Lucknow U...Lucknow Call Girls Service { 9984666624 } ❤️VVIP ROCKY Call Girl in Lucknow U...
Lucknow Call Girls Service { 9984666624 } ❤️VVIP ROCKY Call Girl in Lucknow U...
 
Call Girls Kathua Just Call 8250077686 Top Class Call Girl Service Available
Call Girls Kathua Just Call 8250077686 Top Class Call Girl Service AvailableCall Girls Kathua Just Call 8250077686 Top Class Call Girl Service Available
Call Girls Kathua Just Call 8250077686 Top Class Call Girl Service Available
 
Lucknow Call Girls Just Call 👉👉8630512678 Top Class Call Girl Service Available
Lucknow Call Girls Just Call 👉👉8630512678 Top Class Call Girl Service AvailableLucknow Call Girls Just Call 👉👉8630512678 Top Class Call Girl Service Available
Lucknow Call Girls Just Call 👉👉8630512678 Top Class Call Girl Service Available
 
Chennai ❣️ Call Girl 6378878445 Call Girls in Chennai Escort service book now
Chennai ❣️ Call Girl 6378878445 Call Girls in Chennai Escort service book nowChennai ❣️ Call Girl 6378878445 Call Girls in Chennai Escort service book now
Chennai ❣️ Call Girl 6378878445 Call Girls in Chennai Escort service book now
 
Russian Call Girls In Pune 👉 Just CALL ME: 9352988975 ✅❤️💯low cost unlimited ...
Russian Call Girls In Pune 👉 Just CALL ME: 9352988975 ✅❤️💯low cost unlimited ...Russian Call Girls In Pune 👉 Just CALL ME: 9352988975 ✅❤️💯low cost unlimited ...
Russian Call Girls In Pune 👉 Just CALL ME: 9352988975 ✅❤️💯low cost unlimited ...
 
👉 Chennai Sexy Aunty’s WhatsApp Number 👉📞 7427069034 👉📞 Just📲 Call Ruhi Colle...
👉 Chennai Sexy Aunty’s WhatsApp Number 👉📞 7427069034 👉📞 Just📲 Call Ruhi Colle...👉 Chennai Sexy Aunty’s WhatsApp Number 👉📞 7427069034 👉📞 Just📲 Call Ruhi Colle...
👉 Chennai Sexy Aunty’s WhatsApp Number 👉📞 7427069034 👉📞 Just📲 Call Ruhi Colle...
 

Ospe ophthalmology

  • 1. Objective Structured Practical Examination (OSPE) Md. Anisur Rahman Professor & HOD (Ophthalmology) DMC 7/1/2020 anjumk38dmc@gmail.com 1
  • 2. OSPE: 01 Preparation of Methylprednisolone from this one gram vial with this supplied materials: for the patient of RBN. • One gram Methylprednisolone, • 10 ml distilled water, • 500 mg normal saline, • syringe, • Butterfly needle 7/1/2020 anjumk38dmc@gmail.com 2
  • 3. A Wearing of the gloves 2.0 B Mix 10 ml distilled water in the vial 2,0 C Discard 300 ml normal saline 2,0 D Add this 10 ml medicine with the 200 ml normal saline 2.0 7/1/2020 anjumk38dmc@gmail.com 3
  • 4. OSPE: 02. History taking of R.P • Greetings & self introduction.------------------------------------------0.25 • What is the duration of dimness of vision? --------------------------1.0 • Is the dimness of vision slowly progressive? ---------------------- 1.0 • Was it started with night vision problem? ----------------------------1.0 • Family history of RP. If yes, where they examine? ---------------- 1.0 • H/O consanguine marriage of parent---------------------------------1.0 • History of drug intake -------------------------------------------------- 0.5 7/1/2020 anjumk38dmc@gmail.com 4
  • 5. • History of trauma. --------------------------------------------------0.5 • H/O hearing disorder, ataxia, nystagmus,--------------------- 0.75 • H/O mental retardation ---------------------------------------------1.0 • H/O heart disease? --------------------------------------------------0.5 • H/O hypogenitalism, obesity----------------------------------- 0.75 • H/O diarrhea,, skeletal deformity.--------------------------------- 0.5 • Thanks’ to the patient.-----------------------------------------------0.25 7/1/2020 anjumk38dmc@gmail.com 5
  • 6. OSPE: 3. Your patient need FFA let you counsel the patient Greetings -------------------------------------------------------------0.5 explanation of procedure: • Inj. Na Fluoride -----------------------------------------------------1.0 • Taking of picture ---------------------------------------------------1.0 Prerequisite: • Dilated pupil --------------------------------------------------------1.0 • Renal function test -------------------------------------------------1.0 • Any hypersensitivity of Fluoride --------------------------------1.0 7/1/2020 anjumk38dmc@gmail.com 6
  • 7. Possible side effects: • Nausea/vomiting ------------------------------------------------------ 1.0 • Yellow urine ----------------------------------------------------------- 1.0 • Anaphylaxis /syncope ------------------------------------------------ 1.0 Talk about cost -------------------------------------------------------- 1.0 Thanks & Feedback --------------------------------------------------- 0.5 7/1/2020 anjumk38dmc@gmail.com 7
  • 8. OSPE: 4 Pain that persists for more than one month after other signs and symptoms disappear a) What is the probable diagnosis? b) Which age group suffers more? c) What are the characteristic of pain? Mention 3 d) Why some of the patient suffered from depression? e) What is the most devastating even life-threatening condition? 7/1/2020 anjumk38dmc@gmail.com 8
  • 9. a) Post-herpetic neuralgia-----------------------------------------------2 b) Above 70 year.----------------------------------------------------------1 c) Pain may be • constant or intermittent, ---------------------------------------------1 • worse at night and -----------------------------------------------------1 • aggravated by minor stimuli, touch and heat---------------------1 d) Neuralgia can impair the (QOL), and may lead to depression-1 Patient may commit suicide.--------------------------------------------1 7/1/2020 anjumk38dmc@gmail.com 9
  • 10. OSPE: 5 • During Phacoemulsification of a 55-year-old diabetic patient, nucleus dropped while chopping nucleus. • Write a referral letter to respective specialist for further management: 7/1/2020 anjumk38dmc@gmail.com 10
  • 11. 1) Registration number 1.0 2) Date 0.5 3) Identify of the referred doctor 0.5 x 3 1.5 • Full name • Designation • Address 4) Subject: Dropped nucleus during phaco 1.0 5) Greetings 0.5 7/1/2020 anjumk38dmc@gmail.com 11
  • 12. 6) Patient information 8 x 0.5 4.0 • Name • Age • Statement of the present condition • When nucleus dropped date • Description of dropped nucleus • Vitrectomy done/not done • IOL implanted/not implanted • Present medication receiving 7/1/2020 anjumk38dmc@gmail.com 12
  • 13. 7) Thanks’ 0.5 8) Identity of referring doctor 0.25 x 4 1.0 • Full name • Designation • Address • Signature 7/1/2020 anjumk38dmc@gmail.com 13
  • 14. OSPE: 6 When you are a trainee in a tertiary center you receive a physical assault patient suffering from rupture globe (sclera- corneal injury) right eye. VA is PL+, PR. Now you receive a letter from court to give an injury certificate. write it. 7/1/2020 anjumk38dmc@gmail.com 14
  • 15. 1) Admission Reg No: -----------------------------------------------1.0 2) Date & Time of Examination: ----------------------------------0.50 3) Name of the Patient: --------------------------------------------0.50 4) Age------------------------------------------------------------------0.50 5) Sex------------------------------------------------------------------0.25 6) C/O------------------------------------------------------------------0.50 7) Religion: Islam----------------------------------------------------0.25 7/1/2020 anjumk38dmc@gmail.com 15
  • 16. 8) Occupation-------------------------------------------------------0.25 9) Address----------------------------------------------------------0.25 10)Name of the Identifier----------------------------------------0.50 11)Injury note: -----------------------------------------------------1.0 12)VAR---------------------------------------------------------------0.50 13)VAL---------------------------------------------------------------0.50 7/1/2020 anjumk38dmc@gmail.com 16
  • 17. 14) Slit lamp examination---------------0.25 x 7 ---------------1.75 • Eyelids • Conjunctiva • Cornea • AC • Lens • Pupil • Fundus 7/1/2020 anjumk38dmc@gmail.com 17
  • 18. 15)Nature of weapon----------------------------------------------0.25 16)Nature of trauma-----------------------------------------------0.50 17)Comment---------------------------------------------------------0.25 18)Signature & Date------------------------------------------------0.25 19)(Seal)--------------------------------------------------------------0.25 20)Counter sign------------------------------------------------------0.25 • TOTAL------ 7/1/2020 anjumk38dmc@gmail.com 18
  • 19. OSPE: 7 • A 35 years old patient comes to you for Distance vision and floaters on his Right eye. After taking history ocular and systemic examination and investigations you confirm the patient is suffering from HIV infection and CMV retinitis and CD4 count is <50. You decide to give HAART & Oral Valganciclovir. Counsel the patient. 7/1/2020 anjumk38dmc@gmail.com 19
  • 20. 1) Greetings 2) Give idea about his systemic disease-----------------------------------1.0 3) Give idea about his ocular disease--------------------------------------1.0 4) Explain that his ocular disease is due to his systemic disease-----1.0 5) Needs life style change----------------------------------------------------1.0 6) Needs consultation with physician--------------------------------------1.0 7) Treatment options of systemic disease---------------------------------1.0 8) Treatment option of ocular disease-------------------------------------1.0 7/1/2020 anjumk38dmc@gmail.com 20
  • 21. 9) Duration of treatment----------------------------------------------------0.5 10)Complication of treatment----------------------------------------------0.5 11) Give idea that treatment slow the systemic disease progression but not cure absolutely--------------------------------------------------------0.5 12)Complication if treatment not taken-----------------------------------0.5 13)Follow up and screening------------------------------------------------0.25 14)Recurrence of ocular disease may occur-----------------------------0.25 15)Thanks’---------------------------------------------------------------------0.25 7/1/2020 anjumk38dmc@gmail.com 21
  • 22. OSPE: 8 • Counsel a patient 65 years old, who is suffering from glaucoma and he has also developed cataract with anti- glaucoma eye drop his Intra Ocular Pressure (IOP) is 17 mm of Hg. Counsel the patient regarding your treatment plan 7/1/2020 anjumk38dmc@gmail.com 22
  • 23. 1 Greetings 2 Telling about What is cataract? What is glaucoma? 3 What are the options of management Cataract surgery followed by topical medication Combined phaco trab / Combined SICS trab Trabeculectomy followed by cataract surgery 7/1/2020 anjumk38dmc@gmail.com 23
  • 24. 4 Consequence of only cataract surgery A Post-operative hike of IOP B Medication should be continued for life long C Glaucomatous damage can be progress D Future glaucoma surgery may be difficult E Trab surgery only and its disadvantage F Advantage of combined surgery 5 Counseling about complications of surgeryFeedback from the patient Thanks’ to the patient 7/1/2020 anjumk38dmc@gmail.com 24
  • 25. OSPE: 9A man of 45 year old, refraction done by a junior resident, he done refraction from 1 meter distance & his finding was as below 1) How many mistakes he has been committed? 2) And what are the mistakes 7/1/2020 anjumk38dmc@gmail.com 25
  • 26. A. Four mistakes are there B a) In right axis will be 1800 b) In left eye power will be -1.0 Dsph c) Addition will be + +1.50 d) IPD: 70/66 mm 7/1/2020 anjumk38dmc@gmail.com 26
  • 27. OSPE: 10. Measure IOP with the tonometer with GAT 7/1/2020 anjumk38dmc@gmail.com 27
  • 30. OSPE: 11. Counseling the parents of a child who may need cycloplegic refractive correction (with Atropine 1% eye drop) • Greetings to the parents ------------------------------------------0.25 • Explain the refractive error. --------------------------------------1.0 • Explain the option of treatment. -------------------------------1.0 • Explain what if not treated ------------------------------------- 1.0 • Explain cycloplegic refraction ---------------------------------- 1.0 • Explain the how to apply cycloplegic --------------------------0.5 • Dosage of Atropine-------------------------------------------------0.5 • Explain side effect of cycloplegic & precaution ------------- 1.5 7/1/2020 anjumk38dmc@gmail.com 30
  • 31. • Side effects of Atropine 1% eye drop • Fever • Vision will be blurring for 10 to 14 days • Care & use of spectacles ----------------------------------------------0.75 • Any query from the parents ------------------------------------------0.75 • Regular follow up ---------------------------------------------------------0.75 • Ensure the use of spectacle --------------------------------------------0.75 • Thanks’ --------------------------------------------------------------------- 0.25 7/1/2020 anjumk38dmc@gmail.com 31
  • 32. These two pictures represent two diseases. A. Mention the names of the diseases B. Both the diseases involve different structures of the eyeball but when you get these picture C. In fig: 1, Is it active or chronic stage D. Why vision loss is there (Fig: 1)? Mention 2 OSPE: 12 7/1/2020 anjumk38dmc@gmail.com 32
  • 33. A. Ocular toxoplasmosis & Sarcoidosis --------------------- 4.0 B. When uveal tissue involve ---------------------------------1.5 C. Active-------------------------------------------------------------1.5 D. (Any 2) ---------------------------------------------------------3.0 • macular inflammatory lesions /oedema, • optic nerve involvement • vascular occlusion 7/1/2020 anjumk38dmc@gmail.com 33
  • 34. OSPE: 13 • An old man of 70 year came to you with the complaints of repeated attacks (three times in last two months) of suddenly falls in the ground without warning or loss of consciousness associated with transient, bilateral blurred vision usually lasting a few seconds, sometimes accompanied by flushing lights, ataxia, vertigo, perioral numbness and hemisensory loss associated with visual symptoms. There was no disc oedema. 7/1/2020 anjumk38dmc@gmail.com 34
  • 35. 1) What may be the probable cause? 2) Mention two differential diagnosis 3) Mention one clinical examination & one laboratory investigation to rule out the diagnosis, beyond eye. 4) Two hematological investigations. 5) Two radiological investigations. 7/1/2020 anjumk38dmc@gmail.com 35
  • 36. 1) Vertebro-basilar artery insufficiency ----------------------- 2 2) Any two (1 x 2) ------------------------------------------------ 2 • Giant cell arteritis (GCA) • Ischemic optic neuropathy • Central retinal artery/vein occlusion • Migraine with aura 7/1/2020 anjumk38dmc@gmail.com 36
  • 37. 3) BP in each arm to rule out subclavian steal syndrome-------1 3) ECG/24 hours Holter monitor to rule out dysrhythmia-------1 4) • CBC ----------------------------------------------------------------1 • Serum cholesterol ------------------------------------------------- 1 5) • MRA ------------------------------------------------------------ 1 • Transcranial/vertebral artery Doppler -------------------- 1 7/1/2020 anjumk38dmc@gmail.com 37
  • 38. Vertebrobasilar Insufficiency Aetiopathogenesis: This is a vasculopathic disease affecting the vertebrobasilar arterial supply. It manifests with symptoms due to ischaemia of the brainstem and occipital cortex. Risk factors include diabetes mellitus, hypertension, hyperlipidaemia and cervical spondylosis Ref: Parson’s 518p (22 nd edition) 7/1/2020 anjumk38dmc@gmail.com 38
  • 39. Clinical features: These include:  episodes of transient blurred vision occurring bilaterally,  lasting a few seconds to a few minutes which are  sometimes accompanied by flashing lights. 7/1/2020 anjumk38dmc@gmail.com 39
  • 40. Other associated symptoms  transient diplopia,  ataxia,  vertigo,  dysarthria,  perioral paraesthesia,  dysphasia,  hemiparesis or hemisensory loss. 7/1/2020 anjumk38dmc@gmail.com 40
  • 41. history of The patient may also give a history of drop attacks (sudden episodes of falling to the ground without warning or loss of consciousness). However, the eyes are completely normal on examination 7/1/2020 anjumk38dmc@gmail.com 41
  • 44. Investigation • A complete work-up should be carried out which includes all the tests as for carotid occlusive disease, • ECG and • 24-hour Holter monitoring to rule out sick sinus syndrome and ventricular ectopics, 7/1/2020 anjumk38dmc@gmail.com 44
  • 45. • MRA or transcranial and vertebral Doppler ultrasound to evaluate the posterior cerebral blood flow. • X-rays of the cervical spine to rule out compressive disease of the cervical spine (degenerative changes especially osteophytes encroaching on the arterial foramina) are also required. 7/1/2020 anjumk38dmc@gmail.com 45
  • 46. • A patient of 75-year came to you with the complain of vertigo accompanied by double vision graying of vision, and blurred vision, The patient gave history of suddenly become weak at the knee and crumple a fall that lead to significant head injury. There was no disc edema. 7/1/2020 anjumk38dmc@gmail.com 46
  • 47. 1) What may be the probable cause? And what are the differential diagnoses?------------------------ 7 2) What are the clinical examination & bed side investigation to rule out the diagnosis ------------- 8 3) What are the laboratory investigations you do to rule out the others and why----------------------10 7/1/2020 anjumk38dmc@gmail.com 47
  • 48. OSPE: 14 A lady-60–year-old came to you with the complaints of her eyelash are lengthening, thickening and hyper pigmentation after using an eye drop which you prescribed six months earlier. Overdose of this drop may causes raised of intraocular pressure. 7/1/2020 anjumk38dmc@gmail.com 48
  • 49. 1) What is the name of the eye drop? 2) In which disease it is used? 3) What is the mechanism action of the medicine? 4) What is its dose schedule? 7/1/2020 anjumk38dmc@gmail.com 49
  • 50. 5) Name some drops of this group. 6) If there is no side effect, you have to stop the medication before cataract surgery. Why? 7) Which hyper pigmentation is reversible and which is not 7/1/2020 anjumk38dmc@gmail.com 50
  • 51. 1) Prostaglandin analogue--------------------------------------2.0 2) Glaucoma-------------------------------------------------------1.5 3) Enhancement of uveoscleral aqueous outflow, although increased trabecular outflow facility-----------------------1.5 4) Once in a day---------------------------------------------------1.0 7/1/2020 anjumk38dmc@gmail.com 51
  • 52. 5a) Latanaprost-------------------------------------------------------0.5 5b) Travaprost--------------------------------------------------------0.5 5c) Bimataprost-------------------------------------------------------0.5 5d) Tafluprost----------------------------------------------------------0.5 6) To prevent cystoid macular oedema----------------------------1.0 7) Hyper pigmentation of periocular skin is common but reversible. Irreversible iris hyper pigmentation--------------0.5 + 0.5-----1.0 7/1/2020 anjumk38dmc@gmail.com 52
  • 53. OSPE: 15 A-lady of -25-year old newly married came to you with the complaints of non-specific headache worsening in morning, diffuse type exacerbated by the Valsalva maneuver. Take the relevant history for diagnosis. 7/1/2020 anjumk38dmc@gmail.com 53
  • 57. OSPE: 16 Prepare Injection Amikacin for Intravitreal 7/1/2020 anjumk38dmc@gmail.com 57
  • 60. OSPE: 17 A 7 year child has come to you by his parents with decreased Visual acuity in both eyes. You have noticed that the patient is irritable, with poor self- image. O/E You have found bruises over hand. Ocular examination shows sub conjunctival hemorrhage 7/1/2020 anjumk38dmc@gmail.com 60
  • 61. 1) What may be the diagnosis? 2) Mention 2 posterior segments finding you look for. 3) Mention one systemic laboratory investigation 4) What is the treatment plan? 7/1/2020 anjumk38dmc@gmail.com 61
  • 62. ANSWER & MARKING SCHEME 1) Battered baby syndrome 3 2) Multiple layer retinal hemorrhage and RAP 2 3) MRI of Brain, 2 4) Combined approach of pediatrician, ophthalmologist and neurologist. 3 • TOTAL 10 7/1/2020 anjumk38dmc@gmail.com 62
  • 63. OSPE: 18 Measure tear film breakup time: (supplied materials: Fluorescein strip, slit lamp 7/1/2020 anjumk38dmc@gmail.com 63
  • 67. 1) Comment on the grey scale reading of the field. 2) Is the field reliable? Why? 3) What does 24-2 signify? 4) What is the testing strategy used in this patient? 5) Name 3 causes for such a field defect. 7/1/2020 anjumk38dmc@gmail.com 67
  • 68. 1) The grey scale reading of the right eye field indicates darker tone occupying whole superior and inferior field only sparing central field (Tubular field) 2.0 2) The field is reliable because the fixation loss, false positive and false negative errors are within normal limits. 2.0 3) 24 signify temporal/central 24 degree and 2 indicates on either side of the vertical and horizontal meridian points are tested. 1.5 7/1/2020 anjumk38dmc@gmail.com 68
  • 69. • 4) The strategy used is Swedish interactive testing algorithm standard format. 1.5 • 5) Any 3 3 x 1 3.0 a) Advanced Glaucoma (POAG), b) Advanced RP, c) After PRP, d) CRAO with patent cilioretinal artery • TOTAL 10 7/1/2020 anjumk38dmc@gmail.com 69
  • 70. With SITA strategies, false negatives or false positives over about 15% should probably be regarded as highly significant, and with full-threshold strategies, fixation losses over 20% and false positives or negatives over 33%. In patients who consistently fail to achieve good reliability it may be useful to switch to a suprathreshold strategy or kinetic perimetry. 7/1/2020 anjumk38dmc@gmail.com 70
  • 71. False positives are usually assessed by decoupling a stimulus from its accompanying sound. If the sound alone is presented and the patient still responds, a false positive is recorded. With a high false-positive score the grey scale printout appears abnormally pale. In SITA testing, false positives are estimated based on the response time 7/1/2020 anjumk38dmc@gmail.com 71
  • 72. • False negatives are registered by presenting a stimulus much brighter than threshold at a location where the threshold has already been determined. If the patient fails to respond, a false negative is recorded. A high false-negative score indicates inattention, tiredness or malingering, but is occasionally an indication of disease severity rather than unreliability. The grey scale printout in individuals with high false-negative responses tends to have a clover leaf shape 7/1/2020 anjumk38dmc@gmail.com 72
  • 74. • What is a Humphrey 24-2 and 30-2 visual field test? • 24-2: Measures 24 degrees temporally and 30 degrees nasally and tests 54 points. Used for neuro-ophthalmic conditions and general screening as well as early detection of glaucoma. 30-2: Measures 30 degrees temporally and nasally and tests 76 points. Used for general screening, early glaucoma and neurological conditions 7/1/2020 anjumk38dmc@gmail.com 74
  • 75. OSPE: 20 The patient Mr. X 50 years old chooses this frame for his spectacle. You measure the horizontal IPD of Mr. X by using these instruments, which correspondence with the spect frame 7/1/2020 anjumk38dmc@gmail.com 75
  • 76. 1 By using marker pen, mark the Centre of Bridge of Spectacle on the front surface. 2 Put the frame on wearer’s face. Seat/stand directly in front of the subject at the same level. 3 While the subject is looking at your left eye directly, put the light of pen torch on wearer’s right eye and mark the center of wearer’s right pupil (light reflex at the center of the pupil) on the demo lens with a fine pen. 7/1/2020 anjumk38dmc@gmail.com 76
  • 77. 4 Repeat the same procedure on the wearer’s left eye. 5 Remove the frame, now measure the horizontal distance between the ‘Mark’ on demo lens and center of the bridge of the frame. 6 The distance is measured in millimeter both for right and left lens separately indicates the monocular CD. It is written in the form 33/32. The addition of two numbers should equal to binocular IPD. So the B/O IPD is 65 mm. 7/1/2020 anjumk38dmc@gmail.com 77
  • 78. What will be the visual field if damage occurs in 1, 3, 4 & 5? OSPE: 21 7/1/2020 anjumk38dmc@gmail.com 78
  • 79. 1) Bitemporal hemianopia 2 2) Junctional scotoma 2 3) Upper temporal quadratic defect due to pituitary defect 2 4) Lower temporal quadratic defects due to craniopharyngioma 2 7/1/2020 anjumk38dmc@gmail.com 79
  • 80. OSPE: 22 A lady of 33 year old, using spectacle since childhood, now she came to you by heard the news of miracle can be done with laser, which will give her life easy and she will be get rid of her glass. So, you done some pre LASIK investigations, and result is as followed. 7/1/2020 anjumk38dmc@gmail.com 80
  • 81. • Topography • Topography shows most of the colour in the center of the cornea is hot colour. • Topography also shows: Sim K1 in R/E is 47.50. Sim K2 50.50 • Sim k1 in L/E is 45.00. Sim K2 48.75 7/1/2020 anjumk38dmc@gmail.com 81
  • 82. • Applanation Tonometry: • R/E 15 mm of Hg. L/E 9 mm of Hg • C:D ratio is • 0.7 In R/E and 0.4 in L/E • Schirmer test • 2: 5 mm in B/E Now, you have to counsel the lady about her desires: 7/1/2020 anjumk38dmc@gmail.com 82
  • 83. • Rapport building--------------------------------------------0.25 • Tell about her topography---------------------------------3.0 • Cornea shows thinner in the apical portion so LASIK may not be done • Below 450 µm thickness is contraindicated. • Tell about the Applanation Tonometer---------------------------1.5 • IOP change is significant (diff between 2 eyes) 7/1/2020 anjumk38dmc@gmail.com 83
  • 84. • During the insertion of suction cup IOP may increase above 65 mm of Hg so there may be chance of vision reduction. • C:D ratio is significant------------------------------------------1.0 • So, you may be a patient of POAG • To diagnose the POAG we should do VFA & OCT • POAG is a silent killer of vision. 7/1/2020 anjumk38dmc@gmail.com 84
  • 85. • You are suffering from dry eye, LASIK will also increase dryness----1.0 • You can use RGP contact lens-----------------------------------1.0 • Tell her about the advantage & disadvantage of RGP lens as well as cost-----------------------------1.0 • Other option is clear lens extraction--------------------------------1.0 • Thanks’ to the patient-----------------------------------------------0.25 • TOTAL ---------------------------------------------------------------10 7/1/2020 anjumk38dmc@gmail.com 85
  • 86. OSPE: 23 • It is an idiopathic multisystem disorder. One of the most common systemic associations of uveitis. It can affect essentially any organ system. It more frequently (10:1) affects patients of black than white ethnicity but is more common in colder climates. Female more commonly affected than male peak age is 20-50 years. 7/1/2020 anjumk38dmc@gmail.com 86
  • 87. 1) What is the name of the disease? 2) Name the 3 major organs it affected. 3) Write 2 anterior segment findings 4) Write 3 posterior segment findings 7/1/2020 anjumk38dmc@gmail.com 87
  • 88. 1) Sarcoidosis------------------------------------------- 2.0 2) 3x1-------------------------------------------------------- 3.0 • a) Lung b) Skin c) Eye. 3) Any two----------2x1---------------------------------------2.0 • a) Conjunctival granuloma • b) Lacrimal gland involvement/dry eye • c) Acute or chronic uveitis 7/1/2020 anjumk38dmc@gmail.com 88
  • 89. 4) Any 3-----------------3x1--------------------------------------- 3.0 • a) Periphlebitis • b) Choroidal infiltrates • c) Multifocal Choroiditis • d) Retinal granuloma • e) Peripheral retinal neovascularization • f) Optic nerve involvement. • TOTAL---------------------------------------------------------- 10 7/1/2020 anjumk38dmc@gmail.com 89
  • 90. OSPE: 24 • It is an autoimmune disorder characterized by multisystem inflammation with the generation of autoantibodies. Although the specific cause is unknown. Female at child bearing age is commonly affected. Antibodies (dsDNA) are relatively specific. Renal involvement is common presenting feature. 7/1/2020 anjumk38dmc@gmail.com 90
  • 91. 1) What is the probable diagnosis? 2) Mention 4 hematological test 3) How many patient suffering from ocular involvement? 4) Which one is the most common ocular involvement? 5) What are the most vision threatening complication? Mention 2 7/1/2020 anjumk38dmc@gmail.com 91
  • 92. 1) SLE 2) .Any 4 a) CBC with ESR b) Serum creatinine c) Complement levels d) Liver function tests e) Autoantibody tests 7/1/2020 anjumk38dmc@gmail.com 92
  • 93. • 3) One third • 4) • The most common manifestation is keratoconjunctivitis sicca. The most vision threatening are a) retinal vasculitis and b) optic neuritis/neuropathy. 7/1/2020 anjumk38dmc@gmail.com 93
  • 94. OSPE: 25 (Counseling) • A 50 year-old man with a painful right blind eye. Advice regarding the various options for treating his painful eye including enucleation or evisceration. 7/1/2020 anjumk38dmc@gmail.com 94
  • 95. Suggestion: OSPE: 25 Suggestion: Ascertain the severity of the pain and the patient's desire of keeping an intact globe. If enucleation is contemplated, consider the need for artificial eye service etc. 7/1/2020 anjumk38dmc@gmail.com 95
  • 96. OSPE: 26 (Counseling) • A 63 year-old man returns 4 weeks later for a follow up cataract visit. Refraction shows that a wrong lens has been inserted resulting in a hypermetropic shift. The patient is unhappy that he could neither read nor see distance without glasses. Explain to him what has happened and the various options open to him including lens exchange and contact lenses. 7/1/2020 anjumk38dmc@gmail.com 96
  • 97. Suggestion: OSPE: 26 Find out how inconvenience it is for the patient. Whether the patient was hypermetropic before the operation and what he had been told about the final refraction prior to the operation? Explain the various options open to him. 7/1/2020 anjumk38dmc@gmail.com 97
  • 98. OSPE: 27 (Counseling) A 35 year-old has had two failed corneal graft for heretic corneal disease. The cornea is heavily vascularized. He likes to have another corneal graft but you know there is little of a successful outcome. Explain to him why you think a re-graft is not appropriate. 7/1/2020 anjumk38dmc@gmail.com 98
  • 99. Suggestion: OSPE: 27 Find out why the patient wants a re-graft and his understanding about the success rate. Is he concerned about the cosmetic effect of a leucoma? In which case cosmetic contact lens may be useful.) 7/1/2020 anjumk38dmc@gmail.com 99
  • 100. OSPE: 28 (Counseling) A 69 year-old woman is referred by her GP for possible cataract operation because of very poor vision in both eyes. After examining the patients, you discover that she has minimal cataract and the poor vision is caused by bilateral disciform macular degeneration. Explain to the patient why you think that cataract is inappropriate. 7/1/2020 anjumk38dmc@gmail.com 100
  • 101. Suggestion: OSPE: 28 • Explain to the patient what the problem is and why cataract extraction is unhelpful. Use the analogy of a camera to describe the eye for example: ' The lens of a camera is similar to the lens of your eye and the film within the camera is similar to the back of your eye. When the lens is damaged we can replace it with a new one which is similar to a cataract operation. However, if the film is scratchy, changing the lens may not alter the image captured on the camera. The problem you have is similar to a scratchy film so cataract operation will not improve your vision.' If there is a model of the eye present, use it to illustrate her problem. Suggest blind registration or partial sighted registration and low visual aids.) 7/1/2020 anjumk38dmc@gmail.com 101
  • 102. OSPE: 29 (Counseling) A woman has a 2 year old child with bilateral retinoblastoma. She is planning a second pregnancy and likes to know the risk of a having another child with retinoblastoma. 7/1/2020 anjumk38dmc@gmail.com 102
  • 103. Find out how much the patient knows about the conditions and the inherited patterns. Is her partner aware of her desire of having a second child? Is it possible for the partner to be present during the consultation?) 7/1/2020 anjumk38dmc@gmail.com 103
  • 104. OSPE: 30 (Scenario based) • 71-year-old woman presents with the chief complaint of distorted central vision. Funduscopic examination reveals the presence of subretinal neovascularization and there are depigmented areas in the macula. Distinct yellow-white lesions are seen in the posterior pole surrounding the macula. The patient reports wavy lines during Amsler grid testing. 7/1/2020 anjumk38dmc@gmail.com 104
  • 105. Question 1) What is your probable diagnosis? 2) What are the layers it involves? 3) A highly magnified stereo-examination of the macula is mandatory. What is the clinical examination? 4) What is the most sensitive investigative modality to detect? 5) How he will follow-up at home? 6) When vision is grossly diminished? 7/1/2020 anjumk38dmc@gmail.com 105
  • 106. 1) ARMD 2) choriocapillaris, Bruch’s membrane, retinal pigment epithelium and photoreceptors. 3) The best clinical method is slit-lamp biomicroscopy using a 190 D or 178 D lens 4) OCT (Macular protocol) 7/1/2020 anjumk38dmc@gmail.com 106
  • 107. 5) Amsler grid testing 6) visual impairment may occur associated with a generalized granularity and/or atrophy of the retinal pigment epithelium, photoreceptors and choriocapillaris. 7/1/2020 anjumk38dmc@gmail.com 107
  • 108. OSPE: 31 (Scenario based) • A 50 year old man working at office has been complaining diplopia. He also complaining deglutition difficulty at evening for few days 1) What may be the cause? 2) Name 2 bed side test? 3) What is the confirmatory investigation? 4) What is the role of neuro imaging? 7/1/2020 anjumk38dmc@gmail.com 108
  • 109. 1) Myasthenia gravis 2) sleep test, ice pack test 3) Serum anti ACH-R ab 4) CXR/ CT scan to detect thymoma 7/1/2020 anjumk38dmc@gmail.com 109
  • 110. Sleep test in myasthenia gravis • The diagnosis is usually confirmed by a Tensilon test, which can be complicated by cholinergic side effects that include cardiopulmonary arrest. An alternative, the Sleep test, based on the characteristic of myasthenia that symptoms and signs worsen with fatigue and improve after a period of rest, is safe, moderately sensitive, and specific. The diagnosis of myasthenia can be confirmed by observing resolution of ptosis or ophthalmoparesis immediately after a 30-minute period of sleep; the reappearance of the myasthenic signs over the next 30 seconds to 5 minutes adds further confirmation. 7/1/2020 anjumk38dmc@gmail.com 110
  • 111. OSPE: 32 (Scenario based) A 40 year old man has got admitted in emergency department with history of gun -shot injury one day back. On examination VA right eye NPL, anterior & posterior segment reveals normal(kanski-873) 1) What may be the diagnosis? 2) What is the pathogenesis? 3) What is important clinical finding? 4) What is the treatment? 7/1/2020 anjumk38dmc@gmail.com 111
  • 112. 1) Traumatic optic neuropathy 2) Acceleration of optic nerve at optic canal where it attached to dural sheath due to rupture of micro vascular supply 3) RAPD 4) Pulse dose IV corticosteroid. 7/1/2020 anjumk38dmc@gmail.com 112
  • 113. OSPE: 33 (Scenario based) A 4 year old child with severe bilateral congenital myogenic ptosis with poor elevator function. a) What will be his chin position? b) If amblyopia is there, what may be the commonest reason? c) If Ptosis is unilateral what will be the position of ptotic lid in down gaze in comparison to normal eye? d) What percent of congenital ptosis are associated with the Marcus Gunn jaw-winking phenomenon 5% 7/1/2020 anjumk38dmc@gmail.com 113
  • 114. a) will have a chin up head position b) Refractive error c) Above d) 5% 7/1/2020 anjumk38dmc@gmail.com 114
  • 115. OSPE: 34 (Scenario based) It is the second most common retinal vascular disorder after diabetic retinopathy. a) What is the name of the diseases? b) What is about vision? c) What is the most common site of involvement? And why? d) Sometimes there is persistent poor vision. What is the reason? 7/1/2020 anjumk38dmc@gmail.com 115
  • 116. a) BRVO b) If the central macula is involved consist of the sudden painless onset of blurred vision and metamorphopsia. Peripheral occlusion may be asymptomatic. c) Superotemporal quadrant. The increased incidence in the Superotemporal quadrant is thought to be due to increased arteriovenous crossings in that quadrant. d) Due to chronic macular edema 7/1/2020 anjumk38dmc@gmail.com 116
  • 117. OSPE: 35 (Scenario based) • A sudden painless monocular fall in vision. The pathogenesis is believed to follow the principles of Virchow’s triad for thrombogenesis, involving vessel damage, stasis and hypercoagulability© Age is the most important factor, since over 90% of cases occur in patients over the age of 55 years. a) What is the probable diagnosis? b) What are the risk factors? Mention 4. c) What are the fundus features? 4 7/1/2020 anjumk38dmc@gmail.com 117
  • 118. a) CRVO b) Risk Factors (any: 4) • Age • Hypertension • Hyperlipidaemia • Diabetes mellitus • Oral contraceptive pill • Raised intraocular pressure • Smoking 7/1/2020 anjumk38dmc@gmail.com 118
  • 119. c) i. Tortuosity and dilatation of all branches of the central retinal vein, ii. dot, blot and flame haemorrhages iii. cotton wool spots, iv. optic disc and macular oedema mild 7/1/2020 anjumk38dmc@gmail.com 119
  • 120. OSPE: 36 What will be the image characteristic when object is at 2f1: (Convex Lens) a) Location b) Orientation c) Size d) Type of image 7/1/2020 anjumk38dmc@gmail.com 120
  • 121. a) Location: At 2 f2 b) Orientation: inverted c) Size: Same d) Type of image: Real 7/1/2020 anjumk38dmc@gmail.com 121
  • 122. 7/1/2020 anjumk38dmc@gmail.com 122 OSPE: 37 Write down the retinoscope findings of the following glass from 67 cm distance OD: -0.75 Dsph/-0.75Dcyl 180 OS: Plano/-1.00 Dcyl 180
  • 123. 7/1/2020 anjumk38dmc@gmail.com 123 Write down the retinoscope findings of the following glass from one meter distance OD: -0.75 Dsph/-0.75Dcyl 180 OS: Plano/-1.00 Dcyl 180
  • 124. OSPE: 38 A patient, who is suffering from cataract came for surgery but uncontrolled diabetes, refer the patient to diabetologist with a referral note
  • 125. ANS: 38 A. Date & time: 29/04/2018 B. Time: 8-30 am. C. To, D. Assistant / Associate professor E. Department of Endocrinology F. DMCH
  • 126. G. Dear Sir. H. With due respect I. I am the undersigned, like to inform you that patient Mr. X, 67 years old, hailing from Manikganj suffering from bilateral Senile Mature Cataract (B/E) he is also a patient of DM last 20 years, at present his blood sugar (2 hours ABF) is 15 Mmol/dl in spite of taking insulin regularly. He needs cataract surgery but uncontrolled diabetes is contraindicated for his surgery.
  • 127. • So, I will be obliged if your kind enough to give your valuable opinion regarding his diabetes control J. Thank you K. Signature L, Name of doctor M. Designation N. Institute & Department
  • 128. Marking Scheme: A. Date & time-----------------------------------------------------------------1 B. Addressing: Professor/Assoc prof/ Assitt Prof-------------------------1 C. Dear sir----------------------------------------------------------------------1 • Content: D. Patient identification------------------------------------------------------1 E, From where-----------------------------------------------------------------1
  • 129. F. Patient findings-------------------------------------------------------------1 G. What we want--------------------------------------------------------------1 H. Thank you-------------------------------------------------------------------1 I. Name & designation--------------------------------------------------------1 J. Signature---------------------------------------------------------------------1 TOTAL: -------------------------------------------------------------------10
  • 130. OSPE: 39 A 20-year girl having uneventful DCR surgery under L/A right side of the eye Prepare a discharge certificate for the patient.
  • 131. ANS: 39 Parameters Marks A. Identification of the patient Name 0.5 Age 0.25 Gender 0.25 Address 0.5 Mobile No. 0.25
  • 132. B. Operation note Date & time 0.25 Name of surgery 0.25 Indication of surgery 0.25 Name of anesthesia 0.5 Name of surgeon 0.5
  • 133. C. Post-operative findings Condition of Incision area 0.5 Any discharge 0.5 Conjunctiva, eyelid, cornea 0.5 D. Post-operative treatment Systemic antibiotic 1.0 Systemic analgesic 0.5 Anti ulcerant 0.25 Topical antibiotic 0.5
  • 134. E. Advice No water to eye 0.5 Use dark glass 0.25 Regular use of medicine 0.25 Any problem come to doctor 0.25 Follow up 0.25 F. Identification of certificate preparatory Signature with date 0.25 Name of the doctor with designation 0.5 Seal of the department 0.5  TOTAL 10
  • 135. OSPE: 40 A 60 years old male patient having uneventful phacoemulsification with PC- IOL implantation under topical anesthesia in his right eye Prepare a discharge certificate for the patient.
  • 136. ANS: 40 Parameters Marks A. Identification of the patient Name 0.5 Age 0.25 Gender 0.25 Address 0.5 Mobile No. 0.25
  • 137. B. Operation note Date & time 0.5 Name of surgery 0.5 Name of anesthesia 0.5 Name of surgeon 0.5 C. Post-operative findings Visual acuity 1.0 Anterior segment 1.0 Posterior segment 0.5
  • 138. D. Post-operative treatment Topical antibiotic 0.5 Topical steroid 0.5 E. Advice No water to eye 0.25 Use dark glass 0.25 Regular use of medicine 0.25 Any problem come to doctor 0.25 Follow up 0.25
  • 139. F. Identification of certificate preparatory Signature with date 0.5 Name of the doctor with designation 0.5 Seal of the department 0.5 TOTAL 10
  • 140. OSPE: 41 A. Identify the instrument? Who invented it? B. What is function of the sleeve that covers the tip of the internal tube? C. Name 3 important parameters that have to present during surgery
  • 141. ANS: 41 A. a) Phaco hand piece---------------------------------------------------------1 b) Charles David Kelman----------- +0.5 + 0.5 + 1.0 ------------------2 B. It protects the cornea, iris from transmitted heat energy by the probe --------------------------------------------------------------------2
  • 142. C. a) Power ----------------------------------------------------------------------2 b) Vacuum --------------------------------------------------------------------2 c) Flow rate ------------------------------------------------------------------1 TOTAL---------------------------------------------------------------------10
  • 143. These 3 pictures indicate post- operative complication of cataract surgery. What are the names? And why they occur?
  • 144. ANS: 42  Fig: 1.  Sunrise syndrome: -------------------------------------------------------- 1  most commonly occurs due to the misplacement of the superior haptic in the ciliary sulcus, while the inferior one is placed into the capsular bag that allows the IOL to subluxated superiorly. ------------3
  • 145.  Fig: 2.  Sunset syndrome: ---------------------------------------------------------1  May result from undetected anterior capsule rupture extending inferiorly allowing the inferior haptic of PCIOL to escape through the defects? ----------------------------------------------------------------------3
  • 146. Fig: 3 Windshield wiper syndrome-----------------------------------------------1 When the overall size of the IOL is smaller it moves horizontally ---1  TOTAL---------------------------------------------------------------------10
  • 147. OSPE: 43 A. What is the white area around the disc? B. Does it usually create any symptoms? C. What change you may get in visual field? D. Can it disappear? If yes, when?
  • 148. A. Medullated nerve fiber-------------------------------------------------2.5 B. No-------------------------------------------------------------------------2.5 C. There will be scotoma opposite to the nerve------------------------2.5 D. Yes. In optic atrophy or demyelinated diseases---------------------2.5 TOTAL--------------------------------------------------------------------10
  • 149. OSPE: 44 Applying pressure patches and shields with following materials • eye pad, • eye shield, • scissors, • adhesive tape, • alcohol pad.
  • 150. ANS: 44 A. Apply a piece of adhesive tape, about 15 centimeters long, to the eye pad---------------------------------------------------------------------------2 B. Ask the patient to close both eyes----------------------------------------2 C. Clean the forehead and zygoma with an alcohol pad to remove the skin oils. This helps the tape stick to the skin--------------------------2
  • 151. D. Fold one pad half, and position the eye pad diagonally over the closed lids of the affected eye and tape firmly, but gently, to forehead and cheek----2 E. Apply a second and third piece of tape to ensure the pad lies flat----------1 F. Extra protection can be given by taping a shield over the pad --------------1 TOTAL------------------------------------------------------------------------------10
  • 152. 7/1/2020 anjumk38dmc@gmail.com 152 OSPE: 45 This is the feature of a patient right eye, 15 days after pterygium surgery (surgery was done in bare sclera technique followed by MMC –C eye drop)
  • 153. A. What is the feature now? B. What is the most probable cause? C. Mention 2 symptoms D. What grievous complication impending to occur? E. How can you manage it?
  • 154. ANS: 45 A. Thinning of sclera----------------------------------------------------------2 B. Mitomycin-C---------------------------------------------------------------2 C. Severe pain, photophobia-------------------------------------------------2 D. Perforation of sclera-------------------------------------------------------2 E. Patch scleral graft----------------------------------------------------------2 TOTAL--------------------------------------------------------------------10
  • 156. 7/1/2020 anjumk38dmc@gmail.com 156 A.What is the name of the tracing paper? B.What is about RNFL thickness map? C.What is about RNFL deviation map? D.What is about TSNIT map? E.What is about colour code? F. What is your conclusion?
  • 157. 7/1/2020 anjumk38dmc@gmail.com 157 A.This is the printout paper of OCT optic nerve head and RNFL of BE B.RNFL thickness map normal C.RNFL deviation map normal D.TSNIT curve shows well maintained of double hump pattern E.The four quadrants show green colour and clock hour map also shows green colours. F. So suggestive of a normal OCT of ONH and RNFL